1
Ich habe folgende Aufgabe vorliegen: 



Ich kann den Beweis einer "normalen geometrischen Reihe" , also für die Summe von k=0 bis n über a^k nachvollziehen. 

Allerdings komme ich nicht weiter, wenn ein Produkt in der Summe enthalten ist. 

Vielleicht kann mir da jemand helfen. Ich habe leider keine Übungen an der Uni, da ich nichts Mathematisches studiere, würde mich aber dennoch gerne ein wenig in dem Fach vertiefen :) 

Quelle: Die Aufgabe stammt aus dem Buch "Höhere Mathematik 1", 1989 Meyberg, Vachenauer, Springer Lehrbuch

EDIT vom 13.04.2022 um 17:19:

Bisher bin ich soweit gekommen: 




Ich bin mir nun nicht sicher, ob die letzte Umformung überhaupt stimmt. 
Des Weiteren weiß ich nicht, wie ich das Summenzeichen sozusagen "wegkriege". 
Wenn zum Beispiel nur die Summe aus endlich vielen a^k gebildet werden würde, könnte man ja mit der Umformung für die endlich geometrische Reihe weiter vorgehen. 
Da allerdings im Summenzeichen nun ein Produkt aus a^k und b^(n-k) vorkommt, bin ich etwas ratlos. 

Sagt mir bitte ruhig, falls das noch zu ungenau beschrieben ist.

EDIT vom 13.04.2022 um 19:20:

Letzendlich war mein Fehler, dass ich nicht verstanden hab, dass man die Induktionsvoraussetzung für das "zu beweisende" einsetzen kann. 

Außerdem habe ich falsch mit Reihen gerechnet. Danke für die Tipps dazu (vgl. Kommentare). 

Dieses Video hat mir sehr geholfen: https://www.youtube.com/watch?v=KAW0Tq7iaRQ

Der Beweis: 
gefragt

Student, Punkte: 21

 

Ich denke Induktionsanfang ist ok, was hast du im Induktionsscbritt versucht?   ─   mathejean 13.04.2022 um 10:39

1
@mathejean ich kann den Induktionsanfang garnicht erkennen? @migelfranz ist dir die Beweisführung mittels vollständiger Induktion bekannt wenn du nichts mathematisches studierst? Wenn ja schreibe wenigstens einmal sauber den Induktionsanfang und die Induktionsvoraussetzung auf. Lade deinen Versuch dazu hoch dann helfen wir weiter. Erst wenn das stimmt kann man sich an den Induktionsschritt machen.   ─   maqu 13.04.2022 um 13:36

Mit Induktionsanfang ok meinte ich, dass sein Problem hier nicht ist und wir lieber über Induktionsschritt sprechen, weil wenn Induktion unklar, dann dass falsche Aufgabe für Einstieg   ─   mathejean 13.04.2022 um 15:01

1
@mathejean ich dachte mir schon fast das du das meinst, obwohl du einige füllwörter weggelassen hast😜 wir wissen doch aber wie so oft garnicht wo das Problem des Fragers beginnt, ob nun im Induktionsschritt oder generell mit induktion … deswegen doch lieber erst einmal den Anfang sauber von ihm formulieren lassen   ─   maqu 13.04.2022 um 15:16

Danke an euch beide für die schnelle Antwort. Ich habe den Beitrag nochmal bearbeitet und mehr Informationen zu meinem Vorgehen hinzugefügt. Sry für die Umstände.

Ich habe bereits folgendes probiert:
* Mit Potenzregeln umformen
* Mit Regeln für das Summenzeichen umformen (bin mir nicht sicher, ob da die letzte Umformung stimmt, vgl Bild 3)
* Die Summenziechen "auseinanderziehen" (da bin ich mir nicht sicher, ob man das darf)
$$\sum_{k=0}^{n}a^k\cdot b^{n-k}=\sum_{k=0}^{n}a^k\cdot\sum_{k=0}^{n}b^{n-k}$$
=> ich hab nirgends gelesen, dass man es explizit darf, deswegen gehe ich auch hier von einem "dead end" aus.

Weiter bin ich nicht gekommen. Wenn ihr Lust habt, könnt ihr mir gerne einen kleinen "Denkanstoß" für den nächsten Schritt geben. Ich bin euch echt sehr dankbar für das Engagement!

  ─   migelfranz 13.04.2022 um 17:32
Kommentar schreiben
1 Antwort
1
Also du versuchst das richtige zu machen (so umformen, dass man Induktionsannahme verwenden kann), leider ist aber \(\sum_{k=0}^{n+1}a^kb^{n+1-k}\not = \sum_{k=0}^{n+1}a^kb^{n-k}+a^{n+1}b^{n+1-k}\). Beim Summe auseinander ziehen musst du alles unter der Summe so stehen lassen, nur das Ende ändert sich. Hinter der Summe schreibst du das was jetzt zu wenig gerechnet wird und setzt für \(k\) die entsprechenden Werte ein. Das \(k\) lebt nur innerhalb der Summe.
Diese Antwort melden
geantwortet

Student, Punkte: 10.87K

 

Habs jetzt geschafft, vielen Dank!!
  ─   migelfranz 13.04.2022 um 19:18

ui es hat sich ja jetzt anscheinend geklärt ... die Problematik lag aber anscheinend wie mikn aufgezeigt hat "nicht nur" in Induktionsschritt ... auch wenn es "nur" noch um Feinheiten in der mathematischen Notation geht ... aber @migelfranz sehr schön wie du es selbst versuchst hast und deine Lösungsskizze hochgeladen hast, genau dann können wir auch am besten helfen und alle etwaigen Fehler finden ... dafür verteile ich gerne auch Upvotes an Fragen   ─   maqu 13.04.2022 um 19:49

Kommentar schreiben